Difference between revisions of "Euc20191/Sub-Problem 2"
Baihly2024 (talk | contribs) (Created page with "==Problem== What integer <math>a</math> satisfies <math>3<\frac{24}{a}<4</math>?") |
|||
Line 2: | Line 2: | ||
What integer <math>a</math> satisfies <math>3<\frac{24}{a}<4</math>? | What integer <math>a</math> satisfies <math>3<\frac{24}{a}<4</math>? | ||
+ | |||
+ | ==Solution== | ||
+ | If <math>a=6</math>, then <math>\frac{24}{6}=4</math> and if <math>a=8</math>, then <math>\frac{24}{8}=3</math>. So, <math>a</math> has to be between <math>6</math> and <math>8</math>, and therefore, it has to be <math>\boxed{7}</math>. | ||
+ | |||
+ | ~Yuhao2012 |
Latest revision as of 11:32, 13 October 2025
Problem
What integer satisfies
?
Solution
If , then
and if
, then
. So,
has to be between
and
, and therefore, it has to be
.
~Yuhao2012